Sei sulla pagina 1di 10

Number Systems Practice Problems with explanations

powered by www.Gr8AmbitionZ.com -

PROBLEMS ON NUMBER SYSTEMS

with explanations
These questions will prove to be good exercise for some good practice in number systems. Try to do the questions without looking at the solutions. The time allotted is 30 mins. All The Best
Question 1 The sum of , (1) less than 1 , , is :

(2) greater than 1

(3) equal to 1 or less than 1 The right answer is Option (1). + 1/9 + 1/16 ++ 1/n2 Consider :

(4) equal to 1/n2 (n + 2)

= 1/(2 x 2) < 1/(1 x 2) because < 1 Similarly, 1/9 = 1/(3 x 3) < 1/(2 x 3) Since 1/3 < Because < 1/3 1/16 = 1/(4 x 4) < 1/(3 x 4)

.. 1/n2 = 1/(nxn) < 1/(n 1)n Since 1/n < 1/(n 1)

i.e. + 1/9 + 1/16 + + 1/n2 < (1/1 ) + (1/2 1/3) ++ [1/(n 1) 1/n] i.e. + 1/9 + 1/16 + + 1/n2 < 1/1 1/n which is less than 1. Hence, the given sum is less than 1 only.

ww

Answer: (1)

for more materials and practice sets visit us at www.Gr8AmbitionZ.com

w.

+ 1/9 + + 1/n2 < 1/(1 x 2) + 1/(2 x 3) + + 1/[(n 1)n]

www.Gr8AmbitionZ.com

..

Gr

8A

mb

iti

on Z.

co

Number Systems Practice Problems with explanations


powered by www.Gr8AmbitionZ.com -

Question 2 What is the digit in the unit place of 3946252 + 734733 ? (1) 8 (3) 6 The right answer is Option (1). 39
46252

(2) 7 (4) 4 + 73
4733

Divide 46, by 4, we get 2 as remainder. Actual remainder (2) Again dividing (2) of 39
46252 252 252

by 4, we get 0 as remainder. And 9 is an odd digit number. So, the last digit

will be 1.

Last digit of 734733 By dividing 47 by 4, we get 3 or 1 as a remainder. So, the last digit of 734733 will be. Last digit of 3 3 3 = i.e. 7. So, the last digit is 1 + 7 = 8. Answer: (1) Question 3

(3) 18

8A

How many terms, at the minimum of sequence 1, , for their sum to be not less than 3? (1) 16 (2) 17 (4) 20

mb
,

The right answer is Option (1). Grouping the terms as follows

1 + + 1/3 + + 1/5 + 1/6 + 1/7 +

Gr

iti
, ., must be added together
3534

Now, ( 1) odd, when divided by 4 will give 1 or 3 as remainder.

on Z.
by 34 the

First we will find the unit digit of 3946252.

co
www.Gr8AmbitionZ.com

(1) + (1/2) + (1/3 + ) + (1/5 + 1/6 + 1/7 + 1/8) + (1/9 + 1/10 + 1/16) + (1/3 + ) > 2 x , i.e. (1/3 + ) > Similarly (1/5 + 1/6 + 1/7 + 1/8) > 4 x 1/8, i.e. > Similarly (1/9 + 1/10 + + 1/16) > 8 x 1/16, i.e. > If you take 16 terms their sum will be greater than 3. Answer: (1)

ww

Question 4

When we divide 33 by 34 the remainder is a and when we divide 33 remainder is b. What is the value of b a? (1) 0 (2) 32 (3) -32 (4) None of these

for more materials and practice sets visit us at www.Gr8AmbitionZ.com

w.

3435

Number Systems Practice Problems with explanations


powered by www.Gr8AmbitionZ.com The right answer is Option (2). 333435 = (34 1)some even power = 333534 = (34 1)some odd power = Now in first case, remainder a = 1 In second case, remainder b = 33 b a = 33 1 = 32. Answer: (2)

(3) 221 The right answer is Option (3). by 17 13 = 221. Answer: (3)

(4) None of these

ab is always a multiple of 13 and cd is always a multiple of 17 so their product has to be divisible

? (1) 1124695 (3) 1012675

The right answer is Option (1).

The sum of all the factor is given by

Gr

= it means

So, the number is 13 11 5

ww

169 1331 5 = 1124695. Answer: (1)

for more materials and practice sets visit us at www.Gr8AmbitionZ.com

www.Gr8AmbitionZ.com

w.

8A

mb

Question 6 What is the number, the sum of whose factors is equal to

(2) 1123685 (4) None of these

iti

on Z.

Question 5 ab and cd are two two-digit natural numbers. 4b + a = 13k1 and 5d c = 17k2, where k1 and k2 are natural numbers. The largest number that will always divide the product of ab and cd is: (1) 13 (2) 17

co

Number Systems Practice Problems with explanations


powered by www.Gr8AmbitionZ.com Question 7

The right answer is Option (1). Smallest number made of 5s and divisible by 3 is 555. Thus A = Question 8 3762 2362 is divisible by: (1) 560 (3) 775 The right answer is Option (2). 3762 2362 xn yn is always divisible by (x y) xn yn is divisible by (x + y) when n is even. So, 3762 2362 will be divisible by (37 23) x Answer: (2) Question 9 (2) 840 (4) 160 = 185.

(1) 100 (3) 99

8A

Find the smallest value of x such that x! ends with exactly 23 zeroes (2) 98 (4) No such no. exist

The right answer is Option (4).

Gr

If we check that, 99! ends with how many zeroes, then we find that it ends with 22 zeroes

mb

So, 3762 2362 will be divisible by both (37 23) and (37 + 23)

(37 + 23) = 14 60 = 840

iti
www.Gr8AmbitionZ.com

So, Now if we find that 100! ends with how many zeroes then it comes out be 24.

ww

20 + 4 = 24 So, there are no such no. exists with ends with 23 zeroes. Answer: (4)

for more materials and practice sets visit us at www.Gr8AmbitionZ.com

w.

19 + 3 = 22

on Z.

co
Answer: (1)

(3) 216

(4) None of these

A is the smallest integer which when multiplied with 3 gives a number made of 5s only. Sum of the digits of A is B. Sum of the digits of B is C. What is the value of C3? (1) 125 (2) 64

Number Systems Practice Problems with explanations


powered by www.Gr8AmbitionZ.com -

(3) 12 The right answer is Option (4). Since 300 < abc < 400 100a + 10b + c = a3 + b3 + c3 put a = 3 and rearrange, we get 300 + 10b + c = 27 + b + c
3 3 3 3

(4) Data insufficient

(b 10b) + (c c) = 273

Now put b = 6 and 7 (as b and c are both less than equal to 9 and 63 and 73 are the nearest cubes to 273). For b = 7 we get c3 c = 0 and two values of c i.e. 0 and 1. Thus abc = 370 or 371 Hence, abc cannot be uniquely determined. Answer: (4) For b = 6. We get c3 c = 117 and no value of c satisfies this.

(3) 14

8A

Question 11 Two different numbers when divided by the same divisor, left remainder 11 and 21 respectively, and when their sum was divided by the same divisor, then the remainder was 4. Find the divisor. (1) 36 (2) 28 (4) 9

Gr

Let the numbers are x1, x2 when divided by y (divisor), quotients are n & m and remainders are 11 & 21 respectively. Therefore x1 = ny + 11

..(1) ..(2)

x2 = my + 21

Adding (1) and (2), x1 + x2 = (n + m)y + 32 = (n + m)y + 28 + 4

ww

As the remainder given is 4, this is possible if y divides 28. So y can be 14 or 28 but 14 is not possible because in (2) the remainder is 21. Answer : (2)

for more materials and practice sets visit us at www.Gr8AmbitionZ.com

www.Gr8AmbitionZ.com

The right answer is Option (2).

w.

mb

iti

on Z.

co

Question 10 abc is a three-digit whole number so that abc = a3 + b3 + c3. [300 < abc < 400]. Find the sum of a, b & c. (1) 10 (2) 11

Number Systems Practice Problems with explanations


powered by www.Gr8AmbitionZ.com -

(1) 22! + 1 (3) 21! + 2 The right answer is Option (1). According to the Wilsons theorem (P 1)! + 1 is always divisible by P Where P is a prime number. So, we can say that (23 1)! + 1 is divisible by 23 22! + 1 is divisible by 23. Answer: (1) Question 13

(2) 22! 1 (4) None of these

Let N = 103 + 104 + 105 + 106 + 107 + 108 + 109. The sum of the digits of N is: (1) 12 (3) 6 The right answer is Option (4). the sum of the digits is 7. Question 14 (2) 1 (4) 7

Since N is written out as a sum of powers of 10, then N can be written as 1111 111 000, and so Answer: (4)

(1) 13 (3) 11 Let the base be N. then 27 = (123)N

8A

If 27 = 123 and 31 = 133, then 15 = ?

The right answer is Option (4).

Gr

mb
(2) 31 (4) 33

iti
www.Gr8AmbitionZ.com

27 = N2 + 2 N1 + 3 No = N2 + 2N + 3 N2 + 2N 24 = (N + 6) (N 4) = 0 N = 6, 4

Now, 15 in base 4 can be written as (33)4.

w.

But the base of a number system cant be negative. Thus N = 4.

Answer : (4)

ww

for more materials and practice sets visit us at www.Gr8AmbitionZ.com

on Z.

co

Question 12 Which of the following numbers is divisible by 23?

Number Systems Practice Problems with explanations


powered by www.Gr8AmbitionZ.com -

(1) 2 (3) 37 The right answer is Option (4). 333


555

(2) 3 (4) All of these + 555


333

The last digit of 333 odd + odd even

555

will be odd, and the last digit of 555

333

So, the last digit is even, which is divisible by 2.

Both the numbers are individually divisible by 3. So 333555 + 555333 is divisible by 3. Hence this expression is divisible by all three numbers. Answer : (4)

And similarly, both the numbers are divisible by 37, so, 333555 + 555333 is divisible by 37.

37 Question 16 The value of (1.00008)12500 is (1) < 1 (3) > 2 The right answer is Option (3).

(1.00008)12500 = (1 + .00008)12500 = bn

Applying binomial theorem: (a + b)n = nC0 an + nC1 an 1 . b1 + + nCn 1 . a1 . bn 1 + nCn .

Gr

8A

mb
(2) > 1 (4) > 3

iti

Note : If the number consists of same in the multiples of 3, then it is always a multiple of

on Z.
will be odd.

Check it by option.

co
www.Gr8AmbitionZ.com

= 1 + 12500

+.+

= 2 + .+ Question 17

w.

> 2. Answer : (3)

The last two digits of 25 63 are :

63

25

ww

(1) 35 (3) 55

(2) 75 (4) 45

The right answer is Option (2). Last two digits of 2563 are always 25.

for more materials and practice sets visit us at www.Gr8AmbitionZ.com

Question 15 The expression 333555 + 555333 is divisible by:

Number Systems Practice Problems with explanations


powered by www.Gr8AmbitionZ.com Last two digits of 6325 = an even no 3

even). Thus, last two digits of 25 63 = _ _ _ 2 5 an even no 3 = 75. Answer : (2) Question 18
63 25

The remainder when 1010 + 10100 + 101000 + + 1010000000000 is divided by 7 is: (1) 0 (3) 2 The right answer is Option (4). There are total of 10 terms. Now apply congruence modulo theorem. 10 3 (mod 7) 10 4 (mod 7) 1010 = (4)10 (mod 7) = 410 (mod 7) Now series can be re-written as. 410 + 4100 + 41000 + . + 410000000000 Now 43 1 (mod 7) (43)3 4 + (43)33 4 + . + (43)333333333 4 (2) 1 (4) 5

Thus, remainder when above series is divided by 7 is

40 5 (mod 7). Answer : (4) Question 19

The number 6n2 + 6n for natural n is always divisible by

Gr

(1) 6 only (3) 12 only

8A

[(1) 4 + (1) 4 + . 10 times] = 40

mb
(2) 18 (4) 6 and 12

iti
Answer: (4) (2) an infinite nonrepeating decimal (4) an infinite repeating decimal

on Z.
www.Gr8AmbitionZ.com

The right answer is Option (4). 2.

Since the above expression contains a 6, it is always divisible by 6. Also, n (n + 1) is divisible by

Hence, 6n (n + 1) is divisible by 6 2 = 12. The correct answer is 12 and 6. Question 20

ww

The number is (1) a finite number (3) equal to 1.7320508 The right answer is Option (2).

for more materials and practice sets visit us at www.Gr8AmbitionZ.com

w.

co

Tens digit of any power of a number, whose unit digit is odd and Tens digit is even, is always

Number Systems Practice Problems with explanations


powered by www.Gr8AmbitionZ.com An infinite nonrepeating decimal. Direct from definition. Answer: (2) Question 21 Let D be a decimal of the form D = 0. abcd abcd abcd, where digits a, b, c and d are integers lying between 0 and 9. At most three of these digits are zero. By what number D be multiplied so that the result is a natural number? (1) 999 (3) 49995 The right answer is Option (3). D= , (2) 9990 (4) None of these

Also 49995 = 9999 5

Hence, once we multiply D with 49995 we will get a natural number. Question 22 N = (11111111)2. What is the sum of the digits of N? (1) 72 (3) 64 The right answer is Option (3). 12 = 1, 112 = 121, 1112 = 12321 (2) 62

(4) None of these

111111112 = 123456787654321. The sum of these numbers is 64. Question 23

mb
(2) 1923 (4) 1922

iti
Answer: (3)

What is the remainder when 192319241925 is divided by 1924? (1) 1922 (3) 1

The right answer is Option (3). Remainder = (1)x = 1. Question 24

192319241925 = (1924-1)some evenpower = x

Gr

8A

N = 2 4 6 8 10 . 100. How many zeroes are there at the end of N? (1) 24 (3) 12

(2) 13 (4) None of these

The right answer is Option (3).

ww

N = 2 50!. The highest power of 5 in 50! is 12, therefore there will be 12 zeroes at the end of N. Answer: (3)

50

for more materials and practice sets visit us at www.Gr8AmbitionZ.com

www.Gr8AmbitionZ.com

Answer: (3)

w.

on Z.
Answer: (3)

co

Number Systems Practice Problems with explanations


powered by www.Gr8AmbitionZ.com -

10

(3) 7 The right answer is Option (1). be > 5.

(4) 8

Since, four is one of the characters in the given number, the base for that number system must Assume that the base = 5. Then, (4 54) + (0 53) + (1 52) + (3 51) + (2 50) = 2542. So, by estimation, we can say that the base of the number system is 5. Answer: (1)

Gr

8A

mb

Dont forget to LIKE us there for Daily Updates

iti

We are at Facebook

on Z.
www.Gr8AmbitionZ.com

ww

for more materials and practice sets visit us at www.Gr8AmbitionZ.com

w.

www.facebook.com/Gr8AmbitionZ

co

If in a number system, 40132 corresponds to 2542 in decimal, find the base of the number system. (1) 5 (2) 6

Question 25

Potrebbero piacerti anche